LSAT and Law School Admissions Forum

Get expert LSAT preparation and law school admissions advice from PowerScore Test Preparation.

 Administrator
PowerScore Staff
  • PowerScore Staff
  • Posts: 8917
  • Joined: Feb 02, 2011
|
#22898
Complete Question Explanation

Strengthen-SN. The correct answer choice is (D)

In this stimulus the author presents the following conditional reasoning:

To increase profits in a mature market, one can build more rooms or make improvements:
  • ..... ..... ..... ..... ..... ..... Build more rooms

    Increase profits ..... :arrow: ..... ..... or

    ..... ..... ..... ..... ..... ..... Make improvements
In mature market-Balaria, the author tells us, building more rooms is not an option. This would lead us to the following, new conditional diagram:
  • Increase profits ..... :arrow: ..... Make improvements
The question, a rare fill-in-the-blank, basically asks us how we can conclude that Bellarian hotel owners cannot increase their profits. If they can't increase their profits, it must be that making improvements is also an impossibility. The correct answer choice will reflect this.

Answer choice (A): This answer choice does not logically complete the stimulus, because even at full capacity the hotels might be able to make improvements and charge more.

Answer choice (B): Even if the hotels are sited in the most attractive way possible, this still wouldn't rule out the possibility of making improvements, charging more, and increasing profits.

Answer choice (C): While this answer choice provides reason for the hotels to be concerned about profitability, it does not reflect our prephrased answer above, and has little to do with the discussion from the stimulus.

Answer choice (D): This is the correct answer choice, perfectly reflecting our prephrase from the discussion above. If there is no way to increase the level of luxury, this is another way of saying that there is no way to make further improvements. Together with the fact that there is no way to add more rooms, this answer choice logically completes the stimulus, making it impossible for Bellaria to increase profitability.

Answer choice (E): The demographics of the patrons of Ballarian hotels is irrelevant to this stimulus' question of whether profitability can be increased, considering that there are only two ways to accomplish such an achievement. This choice discusses neither capacity nor improvements, and does not logically complete the stimulus.
 MBG13
  • Posts: 28
  • Joined: Mar 04, 2016
|
#27963
I was able to correctly answer this question, but if it hadn't been for the fact that this was part of the Strengthen lesson plan - I wouldn't have identified this as a Strengthen question. What makes this a strengthen question?
User avatar
 Jonathan Evans
PowerScore Staff
  • PowerScore Staff
  • Posts: 726
  • Joined: Jun 09, 2016
|
#27986
Hi, MBG,

Good question. In fact, this question could be coded as a Justify the Conclusion problem, certainly in sync with a Strengthen problem but in fact somewhat stronger and more precise.

Consider the question stem: "Which one of the following logically completes the argument?"

This rather precise question indicates that the credited answer will not only strengthen the argument but will also "logically complete" it, that is the credited response will make it logically valid/provide a condition sufficient to justify the conclusion.

The explanation above discusses thoroughly the underlying mechanics of the problem. I would add only that given that your task is to prove that the "hotel owners cannot increase their profits," you might look at this problem through the contrapositive:

~make improvements :arrow: ~make profits

You need an answer choice that will guarantee that making improvements is impossible thereby guaranteeing profits impossible. Hence, answer (D) is the credited response.
 martinbeslu
  • Posts: 49
  • Joined: Aug 09, 2017
|
#42625
I'm not sure that I understand why answer choice D is better than answer choice B. In real estate in general and hotels specifically, price is based on location. A hotel can charge more in Manhattan than it can in Newark because of it's location. If the existing hotels could not have been sited any more attractively then this strengthens the argument that no improvement could be made to increase profitability.

Answer choice D, however, leaves open the possibility that if the hotel was moved to a better location (improving the hotel without increasing the number of rooms) the profits would increase. It may be true that "even wealthy patrons" are not prepared to pay more for the hotels in less than ideal locations even if the level of luxury was increased but if hotels on the outskirts of town were moved to the city's most desirable locations the patrons would pay more for the rooms even if the level of luxury was already at the highest possible level. Therefore, it seems that answer choice D does not strengthen the argument any more than answer choice B or even answer choice A.
 Adam Tyson
PowerScore Staff
  • PowerScore Staff
  • Posts: 5153
  • Joined: Apr 14, 2011
|
#42683
Too much new info, martinbeslu! Who says that location matters, or that it affects profits? The stimulus doesn't say anything about that, does it? That's the kind of info that should be left out of your analysis completely. That's what you know, or believe, from the "real world", but the LSAT exists in an alternate universe that, while it resembles ours, cannot be relied upon to be exactly like ours. You have to take what you are given, and nothing else.

One more thing, though - the stimulus DOES say that the land use rules prohibit building new hotels anywhere. Unless you are talking about some loophole that would allow the hotel owners to pick up the existing hotels and move them as a whole to another location, claiming that since they just moved the old hotel that they did not, in fact, build anything new. That would be quite the moving project, and quite the loophole! Again, that requires all sorts of new info.

Finally, focus on the claim here. It isn't that they couldn't have been any more profitable if they did something differently in the past. The claim is that they cannot now do anything to increase their profits. We know they cannot build new hotels or increase the capacity of the existing hotels. The only thing missing is that they cannot improve the hotels in any way that would increase profits. That's where answer D is going - even if they improved the hotels and made the rooms, facilities, common areas, etc. better, nobody would be willing to pay any more.

This stimulus does leave out a crucial element - cost. Perhaps the hotels could be improved in ways that drive down costs? More energy efficiency, perhaps? More automation, cutting out labor costs? Those could lead to more profit, even if nobody pays more. In that way, this stimulus is flawed, and this answer choice less than perfect. Good thing the instructions don't tell us to pick perfect answers, right? Just the best answers, and D is far and away the best of the bunch. Answer B does nothing to strengthen the claim that they cannot become more profitable than they currently are, but only deals with what they did in the past that got them to this point.

Look to the future, not to the past!
 nickp18
  • Posts: 20
  • Joined: May 26, 2020
|
#88146
Hi Powerscore team!

I answered this question correctly, but would like to know how to interpret the word "sited" in this stimulus. Does that simply mean where it is located and how it is situated?
User avatar
 Ryan Twomey
PowerScore Staff
  • PowerScore Staff
  • Posts: 141
  • Joined: Mar 04, 2021
|
#88179
Hey Nickp18,

You are correct. Sited in this case means where the hotels are placed. I would trust your gut on defining words you're unsure of when taking the test.

I wish you all of the luck with your studies.

Best,
Ryan

Get the most out of your LSAT Prep Plus subscription.

Analyze and track your performance with our Testing and Analytics Package.